If both Ryan and Yoshio are assigned to the project, which one of the following could be true?

Stefaniggorman on October 29, 2020

Can you please explain?

I narrowed this down to A and B, but couldn't figure out how to get all the conditions to qualify. Can you please show me how to diagram this?

Reply
Create a free account to read and take part in forum discussions.

Already have an account? log in

Emil-Kunkin on January 26 at 08:35PM

We know if R is in, we have an OR block, and this must go in the first two slots, so

O R l/t _

And y would take the last spot. This means M can't be first.